Difference between revisions of "2006 AMC 12A Problems/Problem 4"

m
m
Line 3: Line 3:
 
A digital watch displays hours and minutes with AM and PM. What is the largest possible sum of the digits in the display?
 
A digital watch displays hours and minutes with AM and PM. What is the largest possible sum of the digits in the display?
  
<math> \mathrm{(A) \ } 17\qquad \mathrm{(B) \ } 19\qquad \mathrm{(C) \ } 21\qquad \mathrm{(D) \ } 22</math>
+
<math>\mathrm{(A)}\ 17\qquad\mathrm{(B)}\ 19\qquad\mathrm{(C)}\ 21\qquad\mathrm{(D)}\ 22\mathrm{(E)}\  23</math>
 
 
<math>\mathrm{(E) \ } 23</math>
 
  
 
== Solution ==
 
== Solution ==
 
+
From the [[greedy algorithm]], we have <math>9</math> in the hours section and <math>59</math> in the minutes section. <math>9+5+9=23\Rightarrow\mathrm{(E)}</math>
From the [[greedy algorithm]], we have 9 in the hours section and 59 in the minutes section. <math>9+5+9=23 \Rightarrow \mathrm {(E)}</math>
 
  
 
== See also ==
 
== See also ==
* [[2006 AMC 12A Problems]]
 
 
 
{{AMC12 box|year=2006|ab=A|num-b=3|num-a=5}}
 
{{AMC12 box|year=2006|ab=A|num-b=3|num-a=5}}
  
 
[[Category:Introductory Number Theory Problems]]
 
[[Category:Introductory Number Theory Problems]]

Revision as of 23:33, 27 April 2008

Problem

A digital watch displays hours and minutes with AM and PM. What is the largest possible sum of the digits in the display?

$\mathrm{(A)}\ 17\qquad\mathrm{(B)}\ 19\qquad\mathrm{(C)}\ 21\qquad\mathrm{(D)}\ 22\mathrm{(E)}\  23$

Solution

From the greedy algorithm, we have $9$ in the hours section and $59$ in the minutes section. $9+5+9=23\Rightarrow\mathrm{(E)}$

See also

2006 AMC 12A (ProblemsAnswer KeyResources)
Preceded by
Problem 3
Followed by
Problem 5
1 2 3 4 5 6 7 8 9 10 11 12 13 14 15 16 17 18 19 20 21 22 23 24 25
All AMC 12 Problems and Solutions